You are on page 1of 35

GS Test-17 (12th April 2020)

Answer Key & Explanation

Q1. Answer: (a)


Explanation:
Several religious groups that emerged during the period of the thirteenth to seventeenth-
century criticised the ritual and other aspects of conventional religion and the social order, using
simple, logical arguments. Among them were the Nath panthis, Siddhacharas and Yogis. All of
them advocated renunciation of the world. So, statement 1 is correct.
To them, the path to salvation lay in meditation on the formless Ultimate Reality and the
realisation of oneness with it. To achieve this, they advocated intense training of the mind and
body through practices like yogasanas, breathing exercises and meditation. So, statement 2 is
correct.
These groups became particularly popular among “low” castes. So, statement 3 is
not correct.
Bhakti saint-poets rejected all forms of ritualism, outward display of piety and social differences
based on birth. Most of them even rejected the idea of renunciation and preferred to live with
their families, earning their livelihood like any other person, while humbly serving fellow human
beings in need. A new humanist idea emerged as they insisted that bhakti lay in sharing others’
pain. However, it was not the Nath panthis, Yogis and Siddhas who focussed on sharing
pain. So, statement 4 is not correct.
Therefore, the correct answer is (a).

Q2. Answer: (c)


Explanation: The state legislature occupies a pre-eminent and central position in the political
system of a state. Articles 168 to 212 in Part VI of the Constitution deal with the state
legislature. Accordingly, states may have a bicameral or unicameral setting. Most of the states
have a unicameral system, while others have a bicameral system. At present (2019), only six
states have two Houses (bicameral). These are Andhra Pradesh, Telangana, Uttar Pradesh,
Bihar, Maharashtra and Karnataka. Hence there exists no uniformity in the organisation of state
legislatures. So, Statement 1 is correct.
The Constitution provides for the abolition or creation of legislative councils in states. According
to Article 168 the Parliament can abolish a legislative council (where it already exists) or create
it (where it does not exist), if the legislative assembly of the concerned state passes a resolution
to that effect by special majority. This Act of Parliament is not deemed as an amendment of the
Constitution for the purposes of Article 368 and is passed as an ordinary piece of legislation
(i.e., by simple majority). So, Statement 2 is correct.
Therefore, the correct answer is (c).

Q3. Answer: (d)


Explanation: India, as a country, is tagged with ‘serious’ level of hunger in the Global Hunger
Index (GHI) 2019. Apart from poverty, one of the primary reasons for children being
undernourished in the country is that often their mothers are undernourished, this is because of
low women empowerment in India.
The link between sanitation and under-nutrition is very strong. Districts with low levels of access
to toilets have much higher rates of child undernourishment compared to districts with relatively
high levels of access to toilets.
Because of poor dietary habits and more intakes of carbohydrates, there is a prevalence of
hidden hunger in India. So, all the options given are correct.
Therefore, the correct answer is (d).

Q4. Answer: (c)

Prelim Test Series (2020) – GS Test 17 (12th April 2020)


Explanation:
Anabaena is a genus of filamentous cyanobacteria that exist as plankton. They are known for
their nitrogen-fixing ability, and they form symbiotic relationships with certain plants, such as the
mosquito fern. Anabena is capable of transforming atmospheric nitrogen into ammonium
ions (inorganic compounds usable by plants). More than 90 per cent of all nitrogen fixation is
affected by these organisms, which thus play an important role in the nitrogen cycle. So,
statement 1 is not correct.
Nitrosomonas is a genus of Gram-negative, rod-shaped chemoautotrophic bacteria. This
organism oxidizes ammonia into nitrite as a metabolic process. Nitrosomonas are useful in
bioremediation. They are important in the nitrogen cycle by increasing the availability of nitrogen
to plants while limiting carbon dioxide fixation. The genus is found in soil, freshwater, and on
building surfaces. So, statement 2 is not correct.
Nitrobacter is a genus comprising rod-shaped, Gram-negative, and chemoautotrophic bacteria.
Nitrobacter plays an important role in the nitrogen cycle by oxidizing nitrite into nitrate in soil
and marine systems. Unlike plants, where electron transfer in photosynthesis provides the
energy for carbon fixation, Nitrobacter uses energy from the oxidation of nitrite ions, NO2−, into
nitrate ions, NO3−, to fulfil their energy needs. So, statement 3 is correct.
Therefore, the correct answer is (c).

Q5. Answer: (c)


Explanation:
The Government of India had released the results of the fourth cycle of All India Tiger
Estimation - 2018, on the occasion of International Tiger Day (29th July). The count of tigers in
India has risen to 2967 in 2018, with an increase of nearly 33% since the last Tiger Census of
2014. This is because of the development of next-generation infrastructure and the increase in
forest cover in the country. The 33% rise in tiger numbers is the highest ever recorded between
2014 and 2018 which stood at 21% between 2006 and 2010 and 30% between 2010 and 2014.
The rise in tiger numbers was in conformity with the average annual growth rate of tigers since,
2006. Madhya Pradesh saw the highest number of tigers at 526, closely followed by Karnataka
at 524 with Uttarakhand at the third position with 442 tigers.
In 2010, India and 12 tiger range countries pledged to double the number of tigers by 2022
under the Global Tiger Recovery Program. So, statement 1 is correct.
Amrabad Tiger Reserve spreads over 2,800 sq.kms in the districts of Mahabubnagar and
Nalgonda, Telangana. It is the largest tiger reserve in the country.
Earlier, it was part of 'Nagarjunasagar-Srisailam Tiger reserve' but post bifurcation of Andhra
Pradesh, the northern part of the reserve is vested with Telangana state and renamed as
'Amrabad Tiger Reserve'. So, statement 2 is correct.
Therefore, the correct answer is (c).

Q6. Answer: (d)


Explanation:
The Karkota period of Kashmir is the most significant in terms of architecture. One of the most
important temples is Pandrethan, built during the eighth and ninth centuries C.E. In keeping with
the tradition of a water tank attached to the shrine, this temple is built on a plinth built in the
middle of a tank. Although there was evidence of both Hindu and Buddhist followings in
Kashmir, this temple is a Hindu one, possibly dedicated to Shiva. The architecture of this temple
is in keeping with the age-old Kashmiri tradition of wooden buildings. Due to the snowy
conditions in Kashmir, the roof is peaked and slants slowly outward.
Therefore, the correct answer is (d)

Q7. Answer: (b)


Explanation: As per Article 243D of the Indian Constitution, inserted by the 73rd Constitutional
Amendment Act:

Prelim Test Series (2020) – GS Test 17 (12th April 2020)


• Not less than one third (including the number of seats reserved for women belonging to the
Scheduled Castes and the Scheduled Tribes) shall be reserved for women. So, statement
1 is correct
• Seats shall be reserved out of the total seats for the Scheduled Castes and the Scheduled
Tribes, in every Panchayat in proportion to their population. So, statement 2 is not correct
• Not less than one-third of the total number of seats reserved for SC/ST’s shall be reserved
for women belonging, to the SC/ST’s. So, statement 3 is correct.
• Offices of the Chairpersons in the Panchayats at the village or any other level shall be
reserved for the Scheduled Castes the Scheduled Tribes and women in such manner as the
Legislature of a State may provide.
• The legislature of a State may make provision for reservation of seats in any Panchayat or
offices of Chairpersons in the Panchayats at any level in favour of a backward class of
citizens.
• Not less than one-third of the total number of offices of Chairpersons in the Panchayats at
each level shall be reserved for women. So, statement 4 is correct
Therefore, the correct answer is (b).

Q8. Answer: (d)


Explanation:
Tax Buoyancy simply measures the actual or observed change in tax revenue relative to GDP.
Therefore, Tax Buoyancy=Proportionate change in tax revenue/Proportionate change in GDP.
So, statement 1 is not correct.
A tax is buoyant when revenues increase by more than, say, 1 % for a 1 % increase in GDP.
Tax elasticity is the responsiveness of tax revenue to changes in the tax rate. For example,
how tax revenue changes if the government reduces corporate income tax from 25 % to 22 %
indicate tax elasticity.
The direct tax to GDP ratio touched a 3-year low in FY20. The tax buoyancy fell to an 18-year
low on account of corporation tax rate cuts and slowdown. The Budget assumes a tax buoyancy
of 1.2 for 2020-21 compared to 0.5 (FY20 RE) and 0.8 (FY19) over the last two years. So,
statement 2 is correct.
Tax buoyancy will be highest for direct taxes. As the economy grows fast, the additional income
generated may go to the rich group. A part of that they have to pay to the government in the
form of taxes. So if the GDP growth rate registers high growth, say nine percent, direct income
tax collection will accelerate. Generally, direct taxes are more sensitive to GDP growth rate. So,
statement 3 is not correct.
Therefore, the correct answer is (d).

Q9. Answer: (c)


Explanation:
Gene pool centres refers to areas on the Earth where important crop plants and domestic
animals originated. They have an extraordinary range of the wild counterparts of cultivated plant
species and useful tropical plants.
Major Gene Centres:
1) Mediterranean Gene Centre
2) South American Gene Centre
3) South West and Middle East
4) African Gene Centre
5) South American and Central American gene centre
6) Indian Gene centre
i. Indus
ii. South Asian: Includes Pakistan, Bangladesh, Sri Lanka, Myanmar
7) Central Asian Gene centre
8) East Asia Gene centres

Prelim Test Series (2020) – GS Test 17 (12th April 2020)


Having recourse to genetic material is essential to adapt and improve agriculture in the face of
threats, such as diseases or warming climate that can alter growing conditions. Thus, gene pool
conservation is important for sustainable development.
Therefore, the correct answer is (c).

Q10. Answer: (b)


Explanation:
Escherichia Coli is a bacterium that is commonly found in the gut of humans and other warm-
blooded animals. While most strains are harmless, some species have been linked to intestinal
disease as well as aggravating antibiotic resistance. So, statement 1 is not correct.
Shiga toxin-producing E. coli (STEC) is a bacterium that can cause severe foodborne diseases.
Escherichia Coli infection is usually transmitted through the consumption of contaminated water
or food, such as undercooked meat products and raw milk. So, statement 2 is correct.
Due to the infection, common symptoms include abdominal cramps and diarrhoea, which may
be bloody. Fever and vomiting may also occur. Most patients recover within 10 days, although
in a few cases the disease may become life-threatening.
Recently, the government has commissioned a Rs. 9.3 crore study to assess the microbial
diversity along the entire length of river Ganga and to check antibiotic resistance in it. The
research project aims to indicate the type of “contamination” (sewage and industrial) in the river
and “threat to human health (antibiotic resistance surge)”, and to identify the sources of
Escherichia coli.
The project will be undertaken by scientists at the Motilal Nehru Institute of Technology,
Allahabad; the National Environmental Engineering Research Institute (NEERI), Nagpur; Sardar
Patel Institute of Science & Technology, Gorakhpur, as well as start-up companies, Phixgen and
Xcelris Labs.
Therefore, the correct answer is (b).

Q11. Answer: (d)


Explanation:
India’s cotton production in 2019 is projected as the highest ever: 354 lakh bales. In 2019, India
is expected to be the world’s largest cotton producer, surpassing China in output.
However, India’s productivity (yield per unit area), is much lower than other major cotton-
producing countries, meaning a much larger area is used for cotton production. Indeed,
India’s productivity has been only a third of these countries for over four decades. India's cotton
productivity level is now among the lowest in the world, according to the Cotton Association of
India (CAI). Cotton productivity level in India has increased from around 300 kg/hectare in the
year 2002-03 to near 500 kg/hectare currently. However, it is still among the lowest in the world.
So, statement 1 is correct.
While only one per cent of India’s cotton production is organic, it is the world’s largest
producer, producing 56 per cent of the world’s organic cotton. Madhya Pradesh accounts for
43 per cent of India's and 24 per cent of the world’s organic cotton production. So, statement 2
is correct.
Therefore, the answer is (d).

Q12. Answer: (c)


Explanation:
The first distinct example of proper Mughal architecture inspired by Persian architecture, is the
tomb of Humayun, in Delhi, built by his widow, Bega Begum. This tomb is important for a proper
study of the development of later Mughal architecture. Although Sikander Lodi's tomb is the first
garden-tomb built in India, it is Humayun's tomb which strikes a new note. So, statement 1
is not correct.
The tomb of Humayun was built by the orders of his first wife and chief consort, Empress Bega
Begum (also known as Haji Begum). Construction began in 1565 and was completed in 1572; it
cost 1.5 million rupees, paid entirely by the Empress. So, statement 2 is not correct.

Prelim Test Series (2020) – GS Test 17 (12th April 2020)


Therefore, the correct answer is (c).

Q13. Answer: (c)


Explanation: The criterion, as per the Citizenship Act, 1955, to acquire citizenship by birth in
India is:
Any person born in India, on or after 26th January 1950 but prior to the commencement of the
1986 Act on 1st July 1987, is a citizen of India irrespective of the nationality of his parents. A
person born in India on or after 1st July 1987 is considered as a citizen only if either of his
parents was a citizen of India at the time of the birth.
Furthermore, those born in India on or after 3rd December 2004 are considered citizens of India
only if:
• Both their parents are citizens of India. So, statement 1 is correct, and statement 3 is
correct.
• If one parent is a citizen of India and the other not an illegal migrant at the time of their birth,
so, statement 2 is correct.
While the children of foreign diplomats posted in India and enemy aliens cannot acquire Indian
citizenship by birth. So, statement 4 is not correct.
The children of legal migrants are eligible for citizenship by birth
Therefore, the correct answer is (c).

Q14. Answer: (a)


Explanation:
Under the Indian income-tax law, an individual, who qualifies as an NRI is required to file India
tax returns if the taxable income exceeds the maximum amount not chargeable to income-tax;
for FY20, it is Rs. 2.5 lakh. However, to determine the threshold limit for the obligation to file tax
returns, the taxable income should be computed before giving effect to the deductions under
Chapter VI-A (such as under Section 80C, 80D and so on). So, statement 1 is correct.
The Union Budget for 2020-21 had tightened the provisions on those seeking to escape tax by
exploiting their non-resident status. While earlier it was possible to be classified as a non-
resident by staying out of the country for 183 days or about six months in a year, this has now
been, in effect, enhanced to 245 days. So, statement 2 is not correct.
Budget 2020-21 proposes that the income of an NRI generated in India will be taxed here.
Indian earnings of NRIs such as rental income from property in the country are intended to be
taxed by way of the new provision. So, statement 3 is correct.
Therefore, the correct answer is (a).

Q15. Answer: (a)


Explanation:
Glanders is a contagious and fatal disease of equines, including horses, donkeys and mules. It
is caused by infection due to the bacterium Burkholderia mallei (B. mallei). B. mallei have a
zoonotic potential and have been considered as a potential biological warfare or bioterrorism
agent as it can cause highly fatal disease in humans. So, statement 1 is correct.
There is no such declaration announced by W.H.O. It is a notifiable disease in India since 1899.
Glanders is also endemic in Africa, Asia, the Middle East, and Central and South America. It
has been eradicated from North America and Australia.
Recently the Ministry of Fisheries, Animal Husbandry, and Dairying has released the
National Action Plan for Control and Eradication of Glanders, according to which the existing
guidelines have now been updated to stop the outbreak of glanders in infected states and
prevent the spread of the disease to non-infected states/zones in India. According to the new
plan, the veterinary authority of a state where many adjoining districts have reported the disease
shall take a decision about whether to permit a fair or not.
Equine fairs, congregations, shows, or any equestrian events in which equines from the
‘unorganized sector’ take part, shall not be permitted within a 25-kilometre radius of an area
declared afflicted with Glanders. So statement 2 is not correct.

Prelim Test Series (2020) – GS Test 17 (12th April 2020)


Therefore, the correct answer is (a).

Q16. Answer: (d)


Explanation: The “gas giants” Jupiter and Saturn are mostly hydrogen and helium. These
planets must have swallowed a portion of the solar nebula intact. The “ice giants” Uranus and
Neptune are made primarily of heavier stuff, probably the next most abundant elements in the
Sun – oxygen, carbon, nitrogen, and sulphur. So, statement 1 is not correct.
Apart from these four planets rest of the planets are known as terrestrial planets. This includes
Mercury, Venus, Earth, and Mars. Further, Pluto is classified as a dwarf planet. So, statement 2
is not correct.
Therefore, the correct answer is (d).

Q17. Answer: (b)


Explanation:
Kuchipudi is performed by a group of actors going from village to village known
as kusselvas. Kuchipudi derives its name from Andhra village Kusselavapuri. Thus, it has its
origin from Andhra Pradesh. So, option (a) is correct.
Kathak is the traditional dance form of Uttar Pradesh. Kathak derived its name from the 'Kathika'
means storytellers who recited verses from the epics, with gestures
and music. So, option (b) is not correct.
Bharatanatyam , the origin of this dance, is solo dance performance of the temple dancers or
the devadasis in Tamil Nadu and hence it is referred to as 'Dashiattam'. So, option (c) is
correct
Sattriya dance is the modern form of dance introduced by the Vaishnava saint Shankara deva in
the 15th century A.D in Assam. The art form derives its name from the Vaishnava monasteries
known as Sattras. It is inspired by the Bhakti Movement. So, option (d) is correct.
Therefore, the answer is (b)

Q18. Answer: (a)


Explanation: United Nations Peacekeeping helps countries torn by conflict create conditions for
lasting peace. UN peacekeepers provide security and the political and peace-building support to
help countries make the difficult, early transition from conflict to peace. UN Peacekeeping is
guided by three basic principles: Consent of the parties; Impartiality; and Non-use of force
except in self-defence and defence of the mandate. Peacekeeping is flexible, and over the past
two decades has been deployed in many configurations. There are currently 13 UN
peacekeeping operations deployed on three continents. Today's multidimensional
peacekeeping operations are called upon not only to maintain peace and security but also to
facilitate the political process, protect civilians, assist in the disarmament,
demobilization and reintegration of former combatants; support the organization of
elections, protect and promote human rights and assist in restoring the rule of law. So,
statement 1, 2 and 3 are correct.
Therefore, the correct answer is (a).

Q19. Answer: (a)


Explanation: The Environment Ministry has put forward draft Environmental and Social
Management Framework (ESMF). It is a part of a World Bank funded project called
Enhancing Coastal and Ocean Resource Efficiency Program (ENCORE). So, statement 2 is
not correct.
ENCORE aims to strengthen integrated coastal zone management in all coastal States and
Union Territories of India. It seeks to strengthen coastal resource efficiency and resilience, for
adopting and implementing integrated coastal management approaches. So, statement 1 is
correct.

Prelim Test Series (2020) – GS Test 17 (12th April 2020)


The draft ESMF lays out guidelines for coastal States to adopt when they approve and regulate
projects in coastal zones. It also outlines how environmental and social aspects should be
integrated into the planning, design, implementation of projects.
Therefore, the correct answer is (a).

Q20. Answer: (c)


Explanation:
Three Buddhist dramas have been discovered from the fragments of manuscripts on palm-leaf,
at Turfan. One of them named Sariputraprakarana is ascribed to Ashvaghosa, who was a
contemporary of Kanishka. The drama has nine acts, and its theme is based on the events
which led up to the conversion of the young Maudgalayana and Sariputra by the Buddha. The
drama has a close relationship with the classical type as laid down in the Natyashastra. 
Ashvaghosha, (80 CE, Ayodhya—150CE Peshawar) was a philosopher and poet who was
considered as India’s greatest poet before Kalidasa (5th century) and the father of Sanskrit
drama; he popularized the style of Sanskrit poetry known as kavya.
Kalidasa is the most popular among the Sanskrit playwrights. His three
works Malavikagnimitra, Vikramorvashiya, Shakuntalam, are some of the finest examples of
classical Sanskrit play. Abhijnana Sakuntalam is a beautiful tale of love and romance, the name
literally meaning 'Of Sakuntala who is recognized by a token'. Abhijnanasakuntalam is the first
Indian play ever to be translated into western languages. It is for the very first time translated in
the English language by Sir William Jones in the year 1789.
Bhavabhuti was an 8th-century scholar of India noted for his plays and poetry, written in
Sanskrit. His plays are considered the equal of the works of Kalidasa. He is believed to have
been the court poet of king Yashovarman of Kannauj. Kalhana, the 12th-century historian,
places him in the entourage of the king, who was defeated by Lalitaditya Muktapida, king of
Kashmir, in 736 AD. Mahaviracharita is a play by him based on the early life of Rama, the hero
of the Ramayana.
Ratnavali is a Sanskrit drama about a beautiful princess named Ratnavali, and a great king
named Udayana. It is attributed to the Indian emperor Harsha (606–648).
Therefore, the correct answer is (c).

Q21. Answer: (a)


Explanation: The Vice–President of India, is the ex-officio Chairman of the Rajya Sabha. Unlike
the Speaker (who is a member of the Lok Sabha), the Chairman is not a member of the Rajya
Sabha. But like the Speaker, he too can cast a vote in the case of an equality of votes (Casting
vote). So, statement 1 is correct.
The Vice–President, cannot preside over a sitting of the Rajya Sabha as its Chairman when a
resolution for its removal is under consideration. However, he can be present and speak in the
House and can take part in its proceedings, without voting, even at such a time. So, statement
2 is not correct.
Therefore, the correct answer is (a).

Q22. Answer: (d)


Explanation:
Index of Economic Freedom 2019 is brought out by the Heritage Foundation. It ranked the
Indian economy 129th among 186 countries and was categorized as ‘mostly unfree’. India
scored 40 on a scale of 0-100 against the world average of 58.5. So, statement 1 is not
correct.
Global Economic Freedom Index is brought out by the Fraser Institute. It ranked India 79th
among 162 countries which shows a jump of 11 spots from the last years’ ranking. So,
statement 2 is not correct.
Therefore, the correct answer is (d).

Q23. Answer: (b)

Prelim Test Series (2020) – GS Test 17 (12th April 2020)


Explanation:
The National Mission for Enhanced Energy Efficiency (NMEEE) is one of the eight national
missions under the National Action Plan on Climate Change (NAPCC). NMEEE aims to
strengthen the market for energy efficiency by creating conducive regulatory and policy regime
and has envisaged fostering innovative and sustainable business models to the energy
efficiency sector. The Mission is implemented since 2011.
Initiatives under NMEEE:
NMEEE consist of four initiatives to enhance energy efficiency in energy-intensive industries
which are as follows:
• Perform Achieve and Trade Scheme (PAT) - Implementing a market assisted compliance
mechanism to accelerate the implementation of cost-effective improvements in energy
efficiency in large energy-intensive industries. So, pair 1 is not correctly matched.
• Market Transformation for Energy Efficiency (MTEE) - Accelerating the shift to energy-
efficient appliances in the specific application through innovative measures to make the
products more affordable. So, pair 2 is not correctly matched.
• Energy Efficiency Financing Platform (EEFP) - Facilitating Financial Institutions to invest
in Energy Efficiency Projects and Programmes. So, pair 3 is correctly matched.
• Framework for Energy Efficient Economic Development (FEEED) - Developing fiscal
instruments to leverage financing for Energy Efficiency through risk mitigation.
The Mission seeks to upscale the efforts to unlock the market for energy efficiency which is
estimated to be around Rs.74,000 Crore and help achieve total avoided capacity addition of
19,598 MW, fuel savings of around 23 million tons per year and greenhouse gas emission
reductions of 98.55 million tons per year at its full implementation stage.
Therefore, the correct answer is (b).

Q24. Answer: (c)


Explanation:
Kaveri river Basin’s upper catchment area receives rainfall during summer by the south-west
monsoon and the lower catchment area during the winter season by the retreating north-east
monsoon. It is, therefore, almost a perennial river with comparatively fewer fluctuations
inflow and is very useful for irrigation and hydroelectric power generation. Thus, the Cauvery is
one of the best-regulated rivers, and 90 to 95 % of its irrigation and power production potential
already stands harnessed. The fertile soil and large-scale irrigation facilities have made the
Cauvery delta the granary of South India.
Options (a) and (d) are not perennial river basins, they flow mostly during South-West
monsoon season while hydropower and irrigation potential of Ganga river basin is still high. So,
option (b) is not correct.
Therefore, the correct answer is (c).

Q25. Answer (b)


Explanation:
Bhaskaracharya was one of the leading mathematicians in the 12th century CE. His
book Siddhantha Shiromani is divided into four sections:
Sections dealing with
• Spheres - Goladhyaya
• Mathematics of planets- Grahaganita
• Arithmetic- Lilavati
• Algebra- Beej Ganita
Therefore, the correct answer is (b).

Q26. Answer: (a)


Explanation: Censure motion should state the reasons for its adoption in the Lok Sabha. It can
be moved against an individual minister or a group of ministers or the entire council of ministers.
It is moved for censuring the council of ministers for specific policies and actions. If it is passed

Prelim Test Series (2020) – GS Test 17 (12th April 2020)


in the Lok Sabha, the council of ministers need not resign from the office. So, statement 1 is
not correct.
No-Confidence Motion: Article 75 of the Constitution states that the Council of Ministers shall be
collectively responsible to the Lok Sabha. The Lok Sabha can remove the ministry from office
by passing a No-Confidence Motion. It need not state the reasons for its adoption in the Lok
Sabha. It can be moved against the entire Council of Ministers only. It is moved for ascertaining
the confidence of Lok Sabha in the Council of Ministers. The motion needs the support of 50
members to be admitted. If it is passed by the Lok Sabha, the Council of Ministers must resign
from office. So, statement 2 is correct.
Adjournment motion is introduced in the Parliament to draw the attention of the House to a
definite matter of urgent public importance and needs the support of 50 members to be
admitted. As it interrupts the normal business of the House, it is regarded as an extraordinary
device. It involves an element of censure against the government, and hence Rajya Sabha is
not permitted to make use of this device. The discussion on this motion should last not less than
two hours and thirty minutes. So, statement 3 is not correct.
Therefore, the correct answer is (a).

Q27. Answer: (a)


Explanation:
The deadweight loss is a cost to the society created by market inefficiency when adequate
output is not produced. In strict terms, it represents loss either in consumer surplus or producer
surplus or both on account of adequate output not being produced in the economy. The
imposition of taxes on a product is one such example because the producer passes a portion of
the tax burden to consumers which reduces the amount of output to be produced.
Therefore, the correct answer is (a).

Q28. Answer: (b)


Explanation: The activities of living organisms which may cause a marked effect on the survival
of others are called biotic interactions. The interactions may be food, shelter or substratum.
Similarly, it may be intra-specific or inter-specific. All the interactions are divided into three
types. They are; Positive interactions, Negative interactions and Neutral interactions.
1. Mutualism: The interactions between two different species in which both the species are
benefited from each other is called mutualism. Lichens are the joint form of algae and fungi.
The algae prepare their own food and supply it to fungi while the fungi provide support to the
algae.
2. Commensalism: It is a positive inter-specific interaction, in which a smaller member called
'commensal', is benefitted, while the larger member called 'host', is neither benefitted nor
harmed. Epiphytes are small green plants found growing on other plants for space only.
They absorb water and minerals from the atmosphere by their hygroscopic roots and
prepare their own food. The plants are not harmed in any way. Examples: Orchids, lianas
and Vanda hanging mosses are common epiphytes found on the tree of tropical rain forests.
3. Parasitism: It is a type of antagonistic inter-specific interaction in which smaller partner,
called the parasite, derives food and shelter from in or on the body of larger partner, called
host, which inhibits the survival of the host. The host can survive without the parasite, but
the parasite cannot survive without the host. Parasitism can be rightly explained as weaker
attacking the stronger. Example: lice on humans, ticks on dogs and copepods on fish.
4. Amensalism: It is an antagonistic inter-specific interaction in which one species is inhibited
while other species is neither benefited nor harmed. In simple words, in amensalism, one
organism does not allow other organisms to grow or live near it. It is also called antibiosis,
and the affected species is called a mental and the affecting species is called an inhibitor.
Such inhibition is achieved through the secretion of certain chemicals called allochemics
or allelopathic substances.
Therefore, the correct match is (b).

Prelim Test Series (2020) – GS Test 17 (12th April 2020)


Q29. Answer: (b)
Explanation:
Indian Seasonal Energy Efficiency Ratio (ISEER) is an evolved rating methodology for air
conditioners that factors in variance in higher temperature in India and rates air conditioners
accordingly. ISEER measures the energy efficiency of air conditioners based on a weighted
average of the performance at outside temperatures between 24 and 43 degree C based on
Indian weather data.
Bureau of Energy Efficiency (BEE) to progressively improve the efficiency of Room Air
Conditioners (AC) and transform the market towards better energy efficiency standards has
brought in Star labelling of ACs. So, statement 1 is correct.
The Indian Seasonal Energy Efficiency Ratio (ISEER) as per the new standards will range from
(3.30 - 5.00) for split and (2.70 – 3.50) for window air conditioners, which will be applicable from
1st January 2021 onwards. So, statement 2 is not correct.
Therefore, the answer is (b).

Q30. Answer: (b)


Explanation:
Ratooning is the agricultural practice of harvesting crop by cutting most of the above-ground
portion but leaving the roots and the growing shoot apices intact, to allow the plants to recover
and produce a fresh crop in the next season. This practice is widely used in the cultivation of
crops such as rice, sugarcane, banana and pineapple.
It has become more popular among the sugarcane growers because of the following
advantages:
1. Saving in the cost of production through saving in cost of seeds, land preparation
and labour cost etc.
2. The leftover in the field, especially in the form of fallen leaves, trash, etc., in due course of
time, gets converted into organic matter.
3. Ratoon crops generally come to maturity at least one-month earlier because roots and
shoots of the plant take lesser time to grow than the fresh seed crops.
So, statements 1 and 2 are the correct.
Disadvantages of ratooning:
1. Crop yields are generally lower than plant crop yields, and the yield reduces with the
successive stages of the crop. With the ageing of roots and shoots plant becomes more
susceptible to pest attack.
2. Nitrogenous fertilizer requirement is more as land is not properly prepared/mulched in
farmland using ratooning. So, statement 3 is not the correct.
Therefore, the correct answer is (b).

Q31. Answer: (a)


Explanation:
Bajirao succeeded his father Peshwa Balaji Vishwanath in 1720 and was Peshwa till his death
in 1740. He was responsible for the expansion of Maratha state from a kingdom in Deccan to an
Empire across Western and Central India. With the Maratha victory at Palkhed (March 1728),
the Nizam was forced to recognise Shahu as the sole Maratha monarch with rights to chauth
and sardeshmukhi of the Deccan. So, statement 1 is correct.
In 1738, he defeated a huge Mughal army under the generalship of Nizam and the treaty of
Bhopal that followed in January 1739 ceded to the Peshwa the subah of Malwa and sovereignty
over all lands between the rivers Narmada and Chambal. In these territories, however, the
Marathas did not try to overturn the local power structure and quickly entered into negotiations
with the local zamindars for the payment of yearly tributes. A civilian system of revenue
administration took time to emerge in this newly conquered region, and this was a feature
typical of all Maratha conquests. So, statement 2 is not correct.
Therefore, the correct answer is (a).

Prelim Test Series (2020) – GS Test 17 (12th April 2020)


Q32. Answer: (b)
Explanation: In addition to the budget that contains the ordinary estimates of income and
expenditure for one financial year; various other grants are made by the Parliament under
extraordinary or special circumstances:
• Supplementary Grant: It is granted when the amount authorised by the Parliament through
the appropriation act for a particular service for the current financial year is found to be
insufficient for that year. So, statement 1 is correct.
• Additional Grant: It is granted when a need has arisen during the current financial year for
additional expenditure upon some new service not contemplated in the budget for that year.
• Exceptional Grant: It is granted for a special purpose and forms no part of the current
service of any financial year. So, statement 2 is not correct.
• Excess Grant: It is granted when money has been spent on any service during a financial
year in excess of the amount granted for that service in the budget for that year. It is voted
by the Lok Sabha after the financial year. Before the demands for excess grants are
submitted to the Lok Sabha for voting, they must be approved by the Public Accounts
Committee of Parliament.
Therefore, the answer is (b).

Q33. Answer: (b)


Explanation:

The Bharat Bond Exchange Traded Fund (ETF) is India’s first corporate bond ETF which will
strengthen the corporate debt market. It would consist of baskets of AAA-rated bonds issued by
public sector units. It will track the new index built by NSE. So, statement 1 is correct.
It will have 2 investment options for two different maturity periods of 3 and 10 years. Therefore it
will provide predictable returns and is expected to generate more interest among retail
investors. So, statement 2 is not correct.
Since fund would be investing in AAA-rated bonds, these units will be relatively secured. So,
statement 3 is correct.
There will be a separate fund of fund for investors not having Demat accounts. So, statement 4
is correct.
Therefore, the correct answer is (b).

Q34. Answer: (c)


Explanation:
Mitochondria are known as the powerhouse of the cell. The energy required for various
chemical activities needed for life is released by mitochondria in the form of ATP (Adenosine

Prelim Test Series (2020) – GS Test 17 (12th April 2020)


Triphosphate) molecules. The body uses energy stored in ATP for making new chemical
compounds and for mechanical work. Mitochondria have their own DNA and ribosomes.
Therefore, mitochondria are able to make some of their own proteins. So, pair 1 is correctly
matched.
Ribosomes, which are present in all active cells, are the sites of protein manufacture. The
manufactured proteins are then sent to various places in the cell depending on need, using the
Endoplasmic reticulum. So, pair 2 is not correctly matched.
Golgi bodies; are involved in the storage, modification and packaging of products in vesicles.
In some cases, complex sugars may be made from simple sugars in the Golgi apparatus. The
Golgi apparatus is also involved in the formation of Lysosomes. So, pair 3 is correctly
matched.
Plastids are Small coloured bodies in the cytoplasm. Some of the plastids contain a green
pigment called chlorophyll. These green coloured plastids are called chloroplasts. They
provide green colour to the leaves and are important for photosynthesis in plants. Plastids are
present only in plant cells. There are two types of plastids - chromoplasts (coloured
plastids) and leucoplasts (white or colourless plastids). So, pair 4 is correctly matched.
Therefore, the correct answer is (c).

Q35. Answer: (b)


Explanation:
Geological Survey of India, Department of Meteorology, Government of India, along with the
recently formed National Institute of Disaster Management, have made an intensive analysis of
more than 1,200 earthquakes that have occurred in India in different years in the past, and
based on these, they divided India into the following five earthquake zones:
i. Very high damage risk zone
ii. High damage risk zone
iii. Moderate damage risk zone
iv. Low damage risk zone
v. Very low damage risk zone
Out of these, the first two zones had experienced some of the most devastating earthquakes in
India. Areas vulnerable to these earthquakes are the North-east states, areas to the north of
Darbhanga and Araria along the Indo-Nepal border in Bihar, Uttarakhand, Western
Himachal Pradesh (around Dharamshala) and Kashmir Valley in the Himalayan region and the
Kuchchh (Gujarat). These areas, along with Andaman and Nicobar Islands are located on
seismically active plate boundaries and hence included in the Very High Damage Risk Zone.
Kathiawar Peninsula of Gujarat state is relatively away from the active fault zone and comes
under Moderate damage risk zone.
Therefore, the correct answer is (b).

Q36. Answer: (a)


Explanation:
Secondary succession is a process started by an event such as forest fire, floods, storms,
hurricanes or by human interventions such as overgrazing, agriculture, etc. that reduces an
already established ecosystem to a smaller population of species, and as such secondary
succession occurs on pre-existing soil. In contrast, primary succession usually occurs in a
place lacking soil.
The stages of secondary succession are similar to those of primary succession in that pioneer
species that arrive in the cleared area slowly give way to a community of intermediate species
over many years before a climax community can become established.
So, statement 1 is correct.
Secondary succession is the ecological succession that occurs after the initial succession has
been disrupted, and some plants and animals still exist. It occurs when plants recognize an area
in which the climax community had been disturbed. It is usually faster than primary
succession because the substrate is already present. Primary succession may often require

Prelim Test Series (2020) – GS Test 17 (12th April 2020)


hundreds of years but secondary succession can in a few years or decades. So, statement 2 is
not correct.
Therefore, the correct answer is (a).

Q37. Answer: (a)


Explanation:
At the Battle of Karnal in 1739, Nadir Shah led his army to victory over the Mughals.
Muhammad Shah surrendered, and then both entered the capital Delhi together. The keys to
the capital of Delhi were surrendered to Nadir Shah. So, statement 1 is correct.
Marathas under Bajirao had attacked Delhi in 1737 (a couple of years before the invasion by
Nadir Shah), which had left Mughal empire weakened. They retreated back to central India after
this attack and didn't come face to face with the Persians under Nadir Shah. So, statement 2 is
not correct.
Therefore, the correct answer is (a).

Q38. Answer: (c)


Explanation: Article 14 states that the State shall not deny to any person equality before the
law or the equal protection of the laws within the territory of India. This provision confers rights
on all persons, whether citizens or foreigners.
Equality before law connotes:
• The absence of any special privileges in favour of any person. So, Statement 1 is correct.
• Equal subjection of all persons to the ordinary law of the land administered by ordinary law
courts.
• No person (whether rich or poor, high or low, official or non-official) is above the law.
Equal protection of laws connotes:
• The equality of treatment under equal circumstances (not under all circumstances as it
would also include unequal circumstances), both in the privileges conferred and liabilities
imposed by the laws. So, statement 2 is not correct.
• The similar application of the same laws to all persons who are similarly situated
• Like should be treated alike without any discrimination.
Article 14 prohibits class legislation and not a classification for the legislation. The legislature
can classify persons for legislative purposes so as to bring them under a well-defined class and
classification would be justified unless it is patently arbitrary. So, statement 3 is not correct.
Therefore, the correct answer is (c).

Q39. Answer: (c)


Explanation:
Creeping Inflation is a type of inflation which increases gradually and continually over time.
The rate of creeping Inflation is below 3% annually. A gradual increase in prices increases the
cost of living, but it is beneficial for the economic growth because people tend to buy with
because of the fear of increased price which results in high demand and increase in demand is
good for the health of the economy. Therefore, developed countries seek creeping Inflation and
target for 2% inflation annually.
Walking inflation can also be referred to as Trotting inflation. The prices of goods and
services increase moderately. The rate of walking inflation remains between 3% to 10%. The
occurrence of this type of inflation gives a warning signal to the government to control inflation
before it creates havoc.
Galloping Inflation occurs when the price of the goods increases double-digit but are usually
around 10% or more per annum. It can also be referred to as jumping inflation. Galloping
Inflation is dangerous for the economy of a country, and it affects the middle class and low-
income population the most.
Hyperinflation is a type of inflation where prices increase at very high rates. The prices of
goods and services increase by more than 50 to 60% of a month. Hyperinflation reduces the

Prelim Test Series (2020) – GS Test 17 (12th April 2020)


value of the currency. This type of inflation is rare to occur, and it takes place in stress situations
like war or collapse in aggregate supply. So, option c is correct.
Therefore, the correct answer is (c).

Q40. Answer: (c)


Explanation: Recently, the Defence Research and Development Organisation (DRDO) and
the Indian Institute of Science (IISc) in Bangalore have developed a new bomb detection
device called RaIDer-X. It is an explosives detection device. It can detect up to 20 explosives
from a stand-off distance of two meters. It is developed by the High Energy Material Research
Laboratory (HEMRL) - an arm of the DRDO in Pune and the Indian Institute of Science in
Bangalore. It can also discern bulk explosives even if they have been concealed. The device
has various applications including narcotics, for local police, for customs and other detection
agencies who need to detect various elements which may be explosive or non-explosive in
nature. Most of the recent terrorist attacks had explosives that were made from easily available
ingredients like petrol and gelatin sticks rather than high-end materials and devices like RaIDer-
X are incremental to thwarting the threat from homemade explosives. So option (c) is correct.
Therefore, the correct answer is (c).

Q41. Answer: (b)


Explanation:
The Himalayan Ranges show a succession of vegetation from the tropical to the tundra, with
the change in altitude. Deciduous forests are found in the foothills of the Himalayas. It is
succeeded by the wet temperate type of forests between an altitude of 1,000-2,000 m. Such
succession is seen in Uttarakhand, Arunachal Himalayas and Purvanchal Hills in North East
India. So, statement 1 is correct.
The Western Ghats and the Nilgiris are closer to the tropics; hence temperate vegetation is
found 1,500 m above the sea level, and Tropical evergreen to moist deciduous on the lower
regions of the Western Ghats, especially in Kerala, Tamil Nadu and Karnataka. So, statement
2 is correct.
The vegetation of Ladakh comes under dry temperate to alpine and high alpine zones. Due to
high aridity, the vegetation is sparse. There are scanty patches of grasses and shrubs for
animals to graze. During the summers, fruit trees such as apples, apricots and walnuts bloom.
So, statement 3 is not correct.
The vegetation of Andaman and Nicobar Islands may be broadly classified into
i. Beach forests, iv. Semi-evergreen forests,
ii. Mangrove forests, v. Moist deciduous forests and
iii. Wet evergreen forests, vi. Grasslands.
So, statement 4 is not correct.
Therefore, the correct answer is (b).

Q42. Answer: (d)


Explanation:
Black Hole Tragedy refers to the scene of an incident on June 20, 1756, in which a number of
Europeans were imprisoned in Calcutta (now Kolkata) and many died. The Europeans were the
remaining defenders of Calcutta following the capture of the city by the nawab Sirāj al-Dawlah,
of Bengal, and the surrender of the East India Company’s garrison under the self-proclaimed
governor of Bengal, John Z. Holwell. According to Holwell, 146 people were locked up, and 23
survived. When the news of this incident and that of the British possessions in Calcutta being
taken over by the Nawab reached the company establishment at Madras as well as Britain,
there was widespread outrage and calls for revenge. This led to dispatching of troops under
Colonel Robert Clive to secure back their Bengal possessions which led to the famous Battle of
Plassey.
Therefore, the correct answer is (d).

Prelim Test Series (2020) – GS Test 17 (12th April 2020)


Q43. Answer: (a)
Explanation:
Habeas Corpus is a Latin term which means ‘to have the body of'. It is an order issued by the
court to a person who has detained another person, to produce the body of the latter before it.
The court then examines the cause and legality of the detention. It would set the detained
person free if the detention is found to be illegal. Thus, this writ is a bulwark of individual liberty
against arbitrary detention. So, statement 1 is correct.
The writ of habeas corpus can be issued against both public authorities as well as private
individuals. So, statement 2 is not correct.
It is not issued where the:
• Detention is lawful.
• The proceeding is for contempt of a legislature or a court.
• Detention is by a competent court.
• Detention is outside the jurisdiction of the court. So, statement 3 is correct.
Therefore, the correct answer is (a).

Q44. Answer: (c)


Explanation: The GST council only recommends the compensation to be paid to the states for
the loss in revenue while the final call is taken by the Parliament. So, statement 1 is correct.
Subsequently, the Goods and Services Tax (Compensation to States), 2017 Act was enacted
by the parliament which had the following major provisions:
• Compensation will be provided to a state for a period of five years from the date on which
the state brings its State GST Act into force.
• For the purpose of calculating the compensation amount in any financial year, the year
2015-16 will be assumed to be the base year, from revenue will be projected. The growth
rate of revenue for a state during the five-year period is assumed be 14% per annum. So,
statement 2 is correct.
Additional information :
• The compensation payable to a state has to be provisionally calculated and released at the
end of every two months, also an annual calculation of the total revenue will be undertaken,
which will be audited by the Comptroller and Auditor General of India.
• A GST Compensation Cess may be levied on the supply of certain goods and services, as
recommended by the GST Council, which will be used for compensation purposes.
Therefore, the correct answer is (c).

Q45. Answer: (c)


Explanation: Recently, researchers have used an invasive plant, water hyacinth, to produce
carbon nanoparticles which can be used for detecting a commonly used herbicide pretilachlor.
The nanoparticles were found to be selective and sensitive for the detection of the herbicide.
The water hyacinth without chlorophyll is powdered and heated at 150-degree Celsius to
convert it to carbon dots. When a nanoparticle is less than 10 nanometre, it is known as a dot or
nanodot. Carbon dots give a green fluorescence under UV light. The extremely small oxygen
functional groups on the surface of the dot are responsible for the fluorescence. The
fluorescence intensity increases in the presence of the herbicide. The carbon dot is extremely
sensitive to pretilachlor and could detect even a very small quantity of it. So, statement 1 is
correct.
It is an aquatic plant native to tropical and sub-tropical South America. It is often a highly
problematic invasive species outside its native range. A single plant of water Hyacinth is
capable of duplicating itself every nine days. It is also referred to as the terror of Bengal, given
its effect on the local ecology and lives of the people. It has an effect on irrigation, hydroelectric
generation, and navigation. It also leads to a drastic reduction in fish production, aquatic crops
and an increase in diseases caused by mosquitoes. So, statement 2 is correct.
Therefore, the correct answer is (c).

Prelim Test Series (2020) – GS Test 5 (12th April 2020)


Q46. Answer: (a)
Explanation: Waymo LLC is an American autonomous driving technology development
company. It is a wholly-owned subsidiary of Alphabet Inc. Waymo originated as a project of
Google and became a stand-alone company in December 2016. In April 2017, Waymo started a
limited trial of a self-driving taxi service in Phoenix, Arizona. On December 5, 2018, the service
launched a commercial self-driving car service called "Waymo One"; users in the Phoenix
metropolitan area use an app to request a pick-up.
The Apolong referred to as the Baidu Apollo project is a driverless vehicle developed by Baidu,
Kinglong and a consortium of more than 40 companies. The Level 4 microcirculation Apolong
bus commenced mass production in 2017.
Therefore, the correct answer is (a).

Q47. Answer: (b)


Explanation:
Krushak Assistance for Livelihood and Income Augmentation (KALIA) Scheme of Odisha
was launched to accelerate agricultural prosperity and elimination of poverty in the State.
Benefits provisioned under different components are: financial support of Rs. Twenty-five
thousand per farm family over five seasons will be provided to small and marginal farmers
so that farmers can purchase inputs like seeds, fertilizers, pesticides, labour & other
investments. So, pair 2 is correctly matched.
Mukhya Mantri Krishi Ashirwad Yojana of Jharkhand: Under the scheme, all the small and
marginal farmers of the State, who have arable land up to a maximum of 5 acres, will be given a
grant-in-aid at the rate of Rs. 5000 per acre per year, which will help them reduce their
dependence on loans. So, pair 1 is not correctly matched.
Rythu Bandhu of Telangana provides Investment Support at the rate of Rs. 4,000 per acre per
season to all the farmers (Pattadars) in the State towards the purchase of various inputs like
seeds, fertilizers etc., as an initial investment before the crop season. So, pair 3 is not
correctly matched.
Therefore, the correct answer is (b).

Q48. Answer: (d)


Explanation:
Paramparagat Krishi Vikas Yojana (PKVY) is an elaborate component of Soil Health
Management (SHM) of the major project “National Mission for Sustainable Agriculture (NMSA)”.
Under PKVY, organic farming is promoted through the adoption of an organic village by cluster
approach and PGS (Participatory Guarantee Systems) certification.
Indiscriminate use of chemical fertilizers in agriculture is a cause of concern and has affected
soil fertility mostly in the states of Punjab, Haryana, and U.P. Salinization and alkalization has
reduced or plateaued agricultural yield. Organic farming involves the use of biological materials,
avoiding synthetic substances to maintain soil fertility and ecological balance, thereby
minimizing pollution and wastage. So, statements 1 and 2 are correct.
Studies from Sikkim (first organic State of India) cites low productivity and the susceptibility of
crops to pest attacks as the reasons for low yield of the crop as compared to synthetic
agricultural practices. In longer run yield may increase due to improved soil fertility but may not
lead to a drastic increase in yield per hectare of farmland. So, statement 3 is not correct.
Under Paramparagat Krishi Vikas Yojana, Marketing support and Certification of the organically
produced crop is done. Adoption of the Participatory Guarantee System (PGS) certification
through the cluster approach is ensured. PGS certification certifies producers based on the
active participation of stakeholders and is built on a foundation of trust, social networks and
knowledge exchange, thus, ensuring the quality of the product. So, statement 4 is correct.
Therefore, the correct answer is (d).

Q49. Answer: (d)


Explanation:

Prelim Test Series (2020) – GS Test 5 (12th April 2020)


The Ramoshi (alternately Berad) is an Indian community found largely in Maharashtra, Madhya
Pradesh, and Karnataka, and are historically associated with great empires, important historical
conflicts and agriculture. They are also described as the Bedar. So, statement a is not correct.
Weavers often belonged to communities that specialised in weaving. Their skills were passed
on from one generation to the next. The tanti weavers of Bengal were one such community who
were famous for their textiles. Similarly, the kaikollars were weavers from south India. So,
statement b and c are not correct.
Some communities like the Agarias specialised in the craft of iron smelting. In the late
nineteenth century, a series of famines devastated the dry tracts of India. In Central India, many
of the Agaria iron smelters stopped work, deserted their villages and migrated, looking for some
other work to survive the hard times. A large number of them never worked their furnaces again.
In April 1904, Charles Weld, an American geologist and Dorabji Tata, the eldest son of Jamsetji
Tata, were travelling in Chhattisgarh in search of iron ore deposits. Agarias helped in finding the
hill whereon exploring the hill, the geologist declared that they had at last found what they had
been looking for. Rajhara Hills had one of the finest ores in the world. So, statement d is
correct.
Therefore, the correct answer is (d).

Q50. Answer: (a)


Explanation: The 44th Constitutional Amendment Act of 1978 abolished the Right to
Property as a Fundamental Right by repealing Article 19(1) (f) and Article 31 from Part III
(Fundamental Rights). Instead, the Act inserted a new Article 300A in Part XII under the
heading ‘Right to Property’. It provides that no person shall be deprived of his property except
by authority of law. Thus, the right to property remains a legal right or a constitutional right. So,
statement 1 is not correct.
The 86th Amendment to the Constitution of India in 2002 provided the Right to Education as a
fundamental right in part-III of the Constitution. A new article 21A was inserted, which made the
right to education a fundamental right for children between 6-14 years. It declares that the State
shall provide free and compulsory education to all children of the age of six to fourteen years in
such a manner as the State may determine. Thus, this provision makes only elementary
education a Fundamental Right and not higher or professional education. So, Statement 2 is
correct.
Therefore, the answer is (a).

Q51. Answer: (c)


Explanation:
The Reserve Deposit Ratio: Banks hold a part of the money people keep in their bank
deposits as reserve money and loan out the rest to various investment projects. Reserve
money consists of two things – vault cash in banks and deposits of commercial banks with RBI.
Banks use this reserve to meet the demand for cash by account holders. Reserve deposit ratio
(RDR) is the proportion of the total deposits of commercial banks kept as reserves. So, pair 1 is
correctly matched.
The currency deposit ratio (CDR) is the ratio of money held by the public in currency to
that they hold in bank deposits. It reflects people’s preference for liquidity. It is a purely
behavioural parameter which depends, among other things, on the seasonal pattern of
expenditure. For example, CDR increases during the festive season as people convert deposits
to cash balance for meeting extra expenditure during such periods. So, pair 2 is correctly
matched.
Therefore, the correct answer is (c).

Q52. Answer: (a)

Prelim Test Series (2020) – GS Test 5 (12th April 2020)


Explanation:

Seagrass can easily be confused with marine macroalgae or seaweed, but there are many
important differences between the two. While seagrasses are considered vascular plants and
have roots, stems and leaves; seaweed is multi-cellular algae and have little or no vascular
tissues. The two differ in reproduction, structure, and how they transport nutrients and dissolved
gases.
The table and diagram below illustrate some of these distinctions.
Kelp is a type of large brown seaweed that grows in shallow, nutrient-rich saltwater, near
coastal fronts around the world. It differs slightly in colour, flavour, and nutrient profile from the
type you may see in sushi rolls. Kelp also produces a compound called sodium alginate.
Therefore, the correct answer is (a).

Q53. Answer: (a)


Explanation:
The Western Dedicated Freight Corridor (WDFC) will begin at Dadri near Delhi and terminate at
the Jawaharlal Nehru Port at Nhava Sheva near Navi Mumbai. It covers 1,504 km across Uttar
Pradesh, Haryana, Rajasthan, Gujarat and Maharashtra state. Thus, it does not pass through
Punjab and Madhya Pradesh.
An inaugural Trial-Run of Indian Railways (IR) freight train on the newly built Rewari-Madar
(Haryana) section of Western Dedicated Freight Corridor (WDFC), was conducted in
December 2019.
Therefore, the answer is (a).

Q54. Answer: (d)


Explanation:
Amazon sailfin catfish is a tropical freshwater fish. It is native to the Amazon River basin of
Brazil and Peru, and it has been destroying fish populations in the wetlands of Kolkata. So,
statement 1 is correct.
Papaya mealybug is a small sap-sucking insect in the mealybug family, Pseudococcidae. It is
found on a number of different hosts, including economically important tropical fruit trees and
various ornamental plants. It is native to Central America. The introduction of this species has
caused damage to papaya cultivation in South India. So, statement 2 is correct.
Senna spectabilis is a deciduous tree native to tropical areas of America. It grows up to 15 to
20 meters in a short period of time and distributes thousands of seeds after flowering. The thick
foliage of the tree arrests the growth of other indigenous tree and grass species. Thus, it causes
food shortages for the wildlife population, especially herbivores. It also adversely affects the
germination and growth of the native species. It is categorized as ‘Least Concern’ under IUCN
Red List. So, statement 3 is correct.
Orange cup coral belongs to a group of corals known as large-polyp stony corals. It is native to
the Indo-Pacific region but has now been reported in the Andaman and Nicobar Islands, the

Prelim Test Series (2020) – GS Test 5 (12th April 2020)


Gulf of Kutch, Kerala, and Lakshadweep as an invasive alien species. So, statement 4 is
correct.
Therefore, the correct answer is (d).

Q55. Answer: (b)


Explanation:
The Charter Act of 1813 incorporated in a significant way all aspirations for change in Britain's
India policy. It renewed the Company's charter for twenty years, and during that period it was
allowed to have its territorial possessions. The Company was to retain the possession of
territories and the revenue for 20 years more, without prejudice to the sovereignty of the Crown.
Thus, the constitutional position of the British territories in India was defined explicitly for the first
time. So, statement a is correct.
The Company was also deprived of its monopoly of trade with India, although its monopoly of
China trade was left untouched for another twenty years. So, statement b is not correct.
And in addition to that, Christian missionaries were henceforth to be allowed to enter India and
preach their religion, subject only to obtaining a licence either from the Court of Directors or the
Board of Control. So, statement c is correct.
Further, Separate accounts were to be kept regarding commercial transactions and territorial
revenues. The power of superintendence and direction of the Board of Control was not only
defined but also enlarged considerably. So, statement d is correct.
Therefore, the answer is (b).

Q56. Answer: (b)


Explanation:
Article 28 distinguishes between four types of educational institutions:
• Institutions recognised by the State. Religious instruction is permitted on a voluntary basis.
So, option (a) is not correct.
• Institutions wholly maintained by the State. Religious instruction is completely prohibited.
So, option (b) is correct.
• Institutions receiving aid from the State. Religious instruction is permitted on a voluntary
basis. So, option (c) is not correct.
• Institutions administered by the State but established under any endowment or trust.
Religious instruction is permitted. So, option (d) is not correct.
Therefore, the correct answer is (b).

Q57. Answer: (b)


Explanation:
The Sustainable Development Goals were proposed in 2012 at the Rio+20 summit. The
Sustainable Development Goals (SDGs), also known as the Global Goals, were adopted by
all United Nations Member States in 2015 as a universal call to action to end poverty,
protect the planet and ensure that all people enjoy peace and prosperity by 2030. So,
statement 1 is not correct.
The SDGs are officially known as ‘Transforming Our World: the 2030 Agenda for Sustainable
Development’. These are a set of 17 aspirational “Global Goals” with 169 targets among them.
So, statement 2 is correct.
The Division for Sustainable Development Goals (DSDG) in the United Nations
Department of Economic and Social Affairs (UNDESA) acts as the Secretariat for the
SDGs, providing substantive support and capacity-building for the goals and their related
thematic issues, including water, energy, climate, oceans, urbanization, transport, science and
technology, the Global Sustainable Development Report (GSDR), partnerships and Small Island
Developing States. DSDG plays a key role in the evaluation of UN system-wide implementation
of the 2030 Agenda and on advocacy and outreach activities relating to the SDGs. So,
statement 3 is correct.
Therefore, the correct answer is (b).

Prelim Test Series (2020) – GS Test 5 (12th April 2020)


Q58. Answer: (d)
Explanation: Die shrink or optical shrink or process shrink refers to the scaling of metal-
oxide-semiconductor (MOS) devices. The act of shrinking a die is to create a somewhat
identical circuit using a more advanced fabrication process. This reduces overall costs for a chip
company, as the absence of major architectural changes to the processor lowers research and
development costs, while at the same time allowing more processor dies to be manufactured on
the same piece of a silicon wafer, resulting in less cost per product sold.
A die is a small block of semiconducting material on which a given functional circuit is
fabricated. Typically, integrated circuits are produced in large batches on a single wafer of
electronic-grade silicon (EGS) or other semiconductors (such as GaAs) through processes such
as photolithography.
Therefore, the correct answer is (d).

Q59. Answer: (b)


Explanation:

In Ocean-Ocean Convergence, a denser oceanic plate subducts below a less dense


oceanic plate forming a trench along the boundary. As the ocean floor crust (oceanic plate)
loaded with sediments subducts into the softer asthenosphere, the rocks in the subduction zone
become metamorphosed (alteration of the composition or structure of a rock) under high
pressure and temperature. After reaching a depth of about 100 km, the plates melt. Magma
(metamorphosed sediments and the melted part of the subducting plate) has a lower density
and is at high pressure. It rises upwards due to the buoyant force offered by the surrounding
denser medium. The magma flows out to the surface. A continuous upward movement of
magma creates constant volcanic eruptions on the ocean floor. Such volcanic landforms all
along the boundary form a chain of volcanic islands which are collectively called as Island

Prelim Test Series (2020) – GS Test 5 (12th April 2020)


Arcs (Indonesian Island Arc or Indonesian Archipelago, Philippine Island Arc, Japanese
Island Arc etc.).

The eastern boundary of the Caribbean Plate is a subduction zone, the Lesser Antilles
subduction zone, where oceanic crust of the South American Plate is being subducted
under the Caribbean Plate. This subduction zone explains the presence of active volcanoes
along the Lesser Antilles. Mount Pelée is an active volcano at the northern end of
Martinique Island (French overseas department) in the Lesser Antilles island arc of the
Caribbean.
So, pair 1 is matched correctly.
Northern arc is formed due to the subduction of the Pacific Plate under the Eurasian Plate.
The trench formed is Japan Trench.
So, pair 2 is matched correctly.
The Nazca Plate, or Nasca Plate, named after the Nazca region of southern Peru, is an
oceanic tectonic plate in the eastern Pacific Ocean basin off the west coast of South America.
The ongoing subduction, along the Peru–Chile Trench, of the Nazca Plate under the South
American Plate is largely responsible for the Andean orogeny.
So, pair 3 is not matched correctly.
Therefore, the correct answer is (b).

Q60. Answer: (d)


Explanation:
The Bastar Rebellion took place in Bastar district, southern Chhattisgarh state, India in 1910.
The rebellion was in reply to perceived exploitation of locals by the British colonial government.
It was mainly organized by forest communities, upon whom the British government had imposed
many regulations.
The Warli revolt of 1945–7 is a critical watershed in the agrarian history of Thane district, and
has had a lasting significance. The demand for land and the demand for higher wages were two
important pivots of the movements led by the Kisan Sabha from the 1940s. Such movements
included struggles against slavery and forced labour, landlords and moneylenders, and for land,
tenancy rights, and forest plots; the issues raised ranged from higher wages for grass-cutting to
the question of the autonomy of the Adivasi people.
The Kol uprising, Kol rebellion, also known in British Indian records as the Kol mutiny was a
revolt of the indigenous Kol people of Chhota Nagpur during 1829-1839 as a reaction to unfair
treatment brought on by the systems of land tenure and administration that had been introduced
by British powers in the area.
The Santhal rebellion, commonly known as Santhal Hool, was a rebellion in present-day
Jharkhand, in eastern India against both the British colonial authority and zamindari system by
the Santhal people. It started on June 30, 1855 and on November 10, 1855 martial law was

Prelim Test Series (2020) – GS Test 5 (12th April 2020)


proclaimed which lasted until January 3, 1856 when martial law was suspended and the
movement was brutally ended by troops loyal to the British. The rebellion was led by the four
Murmu Brothers - Sidhu, Kanhu, Chand and Bhairav.
Therefore, the correct answer is (d).

Q61. Answer: (d)


Explanation: The subordinate courts include the District Judges, Judges of the city civil courts,
Metropolitan magistrates and members of the judicial service of the state. Article 233 of the
Constitution states that:
1) Appointments and promotion of district judges in any state shall be made by the Governor
of the state in consultation with the high court exercising jurisdiction in relation to such
state. So, option (d) is correct.
2) A person not already in the service of the Union or of the state shall only be eligible to be
appointed a district judge if he has been for not less than seven years an advocate or a pleader
and is recommended by the high court for appointment.
Therefore, the correct answer is (d).

Q62. Answer: (c)


Explanation:
The secondary market is the market wherein the trading of securities is done. Secondary market
consists of both equity as well as debt markets.
Securities issued by a company for the first time are offered to the public in the primary
market. Once the Initial Public Offering (IPO) is made, and the stock is listed, they are traded in
the secondary market.
So, statement 1 is not correct.
The main difference between the two is that in the primary market, an investor gets securities
directly from the company through IPOs, while in the secondary market, one purchase securities
from other investors willing to sell the same.
Equity shares, bonds, preference shares, treasury bills, debentures, etc. are some of the
key products available in a secondary market. So, statements 2 and 3 are correct.
Therefore, the correct answer is (c).

Q63. Answer: (c)


Explanation: The ‘SuM4All’ initiative is an umbrella platform that brings together 55 public and
private organizations and companies to act collectively to implement the SDGs and transform
the transportation sector. The Report, released on October 23, 2019, charted a Global
Roadmap for Action (GRA), which provides a catalogue of policy measures that have been used
and tested around the world to achieve four policy goals -
• Universal Access, so no one is excluded from its benefits;
• Efficiency, so scarce resources are well utilized;
• Safety, so transport does not claim lives; and
• Green, so mobility helps check climate change and pollution.
It analysed the mobility performances of 183 countries on these key indicators. The roadmap
presents 182 policy measures across four types of toolboxes:
Regulatory and institutional toolbox, which features strategies and plans for the transport sector
and coordination and cooperation across transport stakeholders;
Engineering and technology toolbox, which includes the design, provision, and maintenance of
transport infrastructure and transport services;
Economics and finance toolbox, which addresses pricing, taxes, subsidies, and cost-
effectiveness;
Communications toolbox, which shares ways to influence behavioural change and promote
knowledge management. The Report recommends that countries select approximately 30 policy
measures from the catalogue and prioritize those measures that have an impact on more than
one policy goal. So statement 1 is correct.

Prelim Test Series (2020) – GS Test 5 (12th April 2020)


The World Bank will perform the secretariat functions of the SuM4All Initiative.
The focus of SuM4All initiative is:
1. Advocacy: Providing thought leadership to influence global and country decision-makers on
transport.
2. Action: Prioritizing the right policies and investments.
3. Financing: Creating and mobilizing finance to achieve sustainable mobility around the globe.
So statement 2 is correct.
Therefore, the correct answer is (c).

Q64. Answer: (b)


Explanation: The Standard Model of particle physics is the theory describing three of the four
known fundamental forces (the electromagnetic, weak, and strong interactions, and not
including the gravitational force) in the universe, as well as classifying all known elementary
particles.
Therefore, the correct answer is (b).

Q65. Answer: (d)


Explanation:
Syed Ahmed Khan, born in a respectable Muslim family, was a loyalist member of the judicial
service of the British government. After retirement in 1876, he became a member of the Imperial
Legislative Council in 1878. His loyalty earned him a knighthood in 1888. So, statement a is
correct.
He wanted to reconcile Western scientific education with the teachings of the Quran which were
to be interpreted in the light of contemporary rationalism and science even though he also held
the Quran to be the ultimate authority. He said that religion should be adaptable with time or
else it would become fossilised, and those religious tenets were not immutable. So, statement
b is correct.
He believed in the fundamental underlying unity of religions or ‘practical morality’. He also
preached the basic commonality of Hindu and Muslim interests. Syed’s progressive social ideas
were propagated through his magazine Tahzeeb-ul-Akhlaq (Improvement of Manners and
Morals). So, statement c is correct.
Active participation in politics at that point, he felt, would invite hostility of the government
towards the Muslim masses. Therefore, he opposed political activity by the Muslims.
Unfortunately, in his enthusiasm to promote the educational and employment interests of the
Muslims, he allowed himself to be used by the colonial government in its obnoxious policy of
divide and rule and, in later years, started propagating divergence of interests of Hindus and
Muslims. So, statement d is not correct.
Therefore, the answer is (d).

Q66. Answer: (d)


Explanation:
NMSA derives its mandate from Sustainable Agriculture Mission which is one of the eight
Missions outlined under National Action Plan on Climate Change (NAPCC).
NMSA will cater to key dimensions of ‘Water use efficiency’, ‘Nutrient Management’ and
‘Livelihood diversification’ through the use of computer applications, by progressively shifting
to environmentally friendly technologies, integrated farming, etc. NMSA aims at promoting
location specific improved agronomic practices through soil health management, enhanced
water use efficiency, judicious use of chemicals, crop diversification and integrated approaches
like crop-sericulture, agro-forestry, fish farming, etc. So, 1, 2 and 3 are the key dimensions of
NMSA.
The focus of NMSA will be to infuse the judicious utilization of resources of commons through a
community-based approach.
Therefore, the correct answer is (d).

Prelim Test Series (2020) – GS Test 5 (12th April 2020)


Q67. Answer: (c)
Explanation: The introduction of Lok Adalats added a new chapter to the justice dispensation
system of this country and succeeded in providing a supplementary forum to the victims for a
satisfactory settlement of their disputes. This system is based on Gandhian principles which
prefer the role of Panchayat/local community in resolving disputes locally, and not procedural
litigation that is costly and partly alien in its roots. It represents Nyaya panchayat in Indian
tradition and helps in establishing grassroots democracy.
It is one of the components of Alternative Dispute Resolution(ADR) systems. It is an Indian
contribution to the world jurisprudence of ADR. Lok Adalat (people’s courts), established by the
government settles the dispute by the principles of justice, equity and fair play, which are the
guiding factors for decisions based on compromises to be arrived at before such Adalats. So,
statements 1 and 2 are correct.
It tends to resolve the cases which are pending or even in the pre-litigation stages. Resolving
disputes through Lok Adalat not only minimizes litigation expenditure, it saves valuable time of
the parties and their witnesses and also facilitates inexpensive and prompt remedy
appropriately to the satisfaction of both the parties. So statement 3 is correct.
Therefore, the correct answer is (c).

Q68. Answer: (b)


Explanation:
There are two groups i.e. Suppliers of money and Holders of money. Supplier of money
includes RBI, Commercial Banks and Central government.
All other entities other than these three are holders of money which are ‘Public’ in this context.
The term ‘Public’ includes: Households, firms, local authorities, NBFCs, non-departmental
PSBs, reserves of foreign banks, governments and IMF etc. held in Indian Rupee held by RBI.
Therefore, the correct answer is (b).

Q69. Answer: (a)


Explanation: A white-yellow star (HD 86081) in Sextans constellation and its Jupiter like exo-
planet (86081b), got Indian names. The International Astronomical Union announced that while
the star will be called Bibhā, the planet will be called Santamasa. The star has been named in
honour of a pioneering Indian woman scientist Dr Bibha Choudhury, who discovered the
subatomic particle pi-meson. Bibha also means ‘a bright beam of light’ in Bengali. The planet
has been named Santamasa to reflect the cloudy nature of its atmosphere. Santamasa is the
Sanskrit term for ‘clouded.’
Therefore, the correct answer is (a).

Q70. Answer: (d)


Explanation:
The mountainous nature of Japan and parts of mainland eastern Asia support little agricultural
activity [80 % land in Japan is classed as ‘non-agricultural’. Around 50% of the total land is
covered by forests]. Japan is not well endowed with natural resources. So, Japan has to take to
the sea. So, statement 1 is correct.
The scarcity of meat (there is little pasture in Japan for livestock farming of any kind)
popularized fish as the principal item of diet and the chief protein food of the Japanese and the
Chinese as well. So, statement 2 is correct.
Gently sloping continental shelves around the islands of Japan are rich in planktons, mainly due
to meeting of warm & cold ocean currents. So, statement 3 is correct.
Fish feeds on minute marine organisms – planktons, which are present in abundance only in
shallow waters adjacent to land masses, where sunlight can penetrate through. Japan also
makes use of fish wastes, fish meals & seaweeds as fertilizers & is among one of the few
countries involved in seaweed cultivation on submerged coastal farms.
Another aspect of Japanese fishing is pearl culture obtained from the shale fish called pearl
oyster found deep inside the sea. There exists a great demand for fish and fish products in the

Prelim Test Series (2020) – GS Test 5 (12th April 2020)


nearby countries where fishing industry in under-developed. Japan has huge stakes in
international fishing enterprises and her advanced fishing techniques give her an edge over her
competitors. Advanced financial services, encouraging government policy, advanced technology
at hand, skilled workforce with decades of experience in fishing and the only available natural
resource to exploit, make Japan a leader in the fishing industry.
Therefore, the correct answer is (d).

Q71. Answer: (b)


Explanation:
India is one of the first countries in the world to develop a comprehensive Cooling Action plan
which has the vision to address the cooling requirement across sectors and lists out actions
which can help reduce the cooling demand. The cooling requirement is cross-sectoral, and an
essential part for economic growth and is required across different sectors of the economy such
as residential and commercial buildings, cold-chain, refrigeration, transport and industries
India Cooling Action Plan (ICAP) would accrue the following benefits to the society over and
above the environmental benefits:
i. Thermal comfort for all – provision for cooling for EWS and LIG housing
ii. Sustainable cooling – low GHG emissions related to cooling
iii. Doubling Farmers Income – better cold chain infrastructure – a better value of products to
farmers, less wastage of produce
iv. Skilled workforce for better livelihoods and environmental protection
v. Make in India – domestic manufacturing of air-conditioning and related cooling
equipment’s.
vi. Robust R&D on alternative cooling technologies – to provide a push to innovation in the
cooling sector.
Therefore, the correct answer is (b).

Q72. Answer: (b)


Explanation:
Gandhi’s idealised state, his Ramrajya did not need a representative government, a constitution,
an army or a police force. Capitalism, communism, exploitation and religious violence would be
absent. Instead, the country was to be modelled on India of the past. In many ways, Gandhi’s
writings call for a pre-modern, morally-enlightened and apolitical Indian state. Swaraj lays stress
on self-governance through individuals and community building.
On the other hand, in his early writings, Bose expressed the opinion that democracy was the
acceptable political system for India. But later, he veered towards the idea that a democratic
system would not be adequate for the process of nation rebuilding. Bose said “You cannot have
a so-called democratic system, if that system has to put through economic reforms on a
socialistic basis. Therefore we must have a political system—a State—of an authoritarian
character....” So, statement 1 is not correct.
Gandhi’s concept of Swaraj had its own brand of economic vision. He wanted a decentralised
economy without state control. Gandhi dismissed both capitalism and Western socialism—the
former for its exploitative excesses and the latter for its connection to industrialisation. He
developed the idea of village Sarvodaya.
Bose considered economic freedom to be the essence of social and political freedom. He was
all in favour of modernisation which was to be brought about by industrialisation. So, statement
2 is correct.
Therefore, the correct answer is (b).

Q73. Answer: (a)


Explanation: The Constitution of India, which made India a sovereign, democratic republic, was
adopted on November 26, 1949, and it came into effect on January 26, 1950, which is
celebrated as Republic Day. Some provisions of the Constitution pertaining to the citizenship,

Prelim Test Series (2020) – GS Test 5 (12th April 2020)


elections, provisional parliament, temporary and transitional provisions and certain short titles
came into force on November 26 1949.
The remaining provisions of the Constitution came into force on the 26 January 1950. This day
is known as the date of commencement of the Constitution.
Therefore, the answer is (a).

Q74. Answer: (b)


Explanation:
A rising share of short-term debt makes the BoP position more vulnerable because of
relatively higher rates of interest on such borrowings. At a time when exports are not growing
rapidly, loans at high-interest rates can create pressure on BoP in the future. So, statement 1
is correct.
A corrected duty structure will reduce the cost of intermediate inputs imported for
manufacturing of exports, thereby making the country’s exports more competitive. The
resulting increase in exports will strengthen India’s BoP position. So, statement 2 is not
correct.
In an open economy framework, a rising fiscal deficit gives rise to two problems. First, it
increases the domestic cost of capital tempting corporates to invest their surplus in the domestic
market while seeking to finance their investment through ECBs. Low-interest rate abroad plus
the cost of hedging turns out to be lower than the domestic cost of capital. Second, as higher
fiscal deficit pushes corporates into seeking larger amounts of foreign savings, the CAD
(Current Account Deficit) widens, bringing the country closer to the twin-deficit challenge,
wherein even a small loss of investor confidence can result in capital flight and a sharp
depreciation of the rupee. In this situation, a default on the BoP becomes highly probable.
So, statement 3 is correct.
An increase in net FPI flows improves the BoP position and arises on account of cross-border
transactions involving debt or equity securities, other than those included in direct investment or
reserve assets. However, FPI is often referred to as “hot money” because of its tendency to flee
at the first signs of trouble in an economy or improvement in investment attractiveness
elsewhere in the world, particularly in the US at the hands of the Federal Reserve. So,
statement 4 is not correct.
Therefore, the correct answer is (b).

Q75. Answer: (b)


Explanation: Although HPV (human papillomavirus) is the most common sexually transmitted
infection (STI), it is very different from HIV and HSV. It is estimated that almost every sexually
active person will get HPV in their lifetime unless they have had the HPV vaccine. There are
many different types of HPV. The virus may not always cause symptoms, but some types can
cause genital warts and certain cancers. So, statement 1 is correct.
Cervical cancer is the second leading cause of cancer deaths among women in India. About
25% of all cervical cancer deaths in the world occur in India. The most common cause of
cervical cancer is HPV. There are more than 100 types of the virus, of which at least 13 are
known to cause cancer. Two strains – type 16 and type 18 – cause about 70% of all cervical
cancer. This makes cervical cancer one of two vaccine-preventable cancers. The other is the
cancer of liver – the Hepatitis B vaccine helps prevent it. So, statement 2 is correct.
The Intensified Mission Indradhanush 2.0 aims to immunize children under 2 years of age and
pregnant women against eight vaccine-preventable diseases. The immunisation drive covers
vaccines for tuberculosis, meningitis, measles, Hepatitis B, tetanus, whooping cough,
poliomyelitis and diphtheria. The vaccines for two other diseases- Hemophilus influenza and
Japanese encephalitis will also be provided under the programme in certain select areas. So,
statement 3 is not correct.
Therefore, the correct answer is (b).

Q76. Answer: (c)

Prelim Test Series (2020) – GS Test 5 (12th April 2020)


Explanation:
An air mass may be defined as a large body of air whose physical properties, especially
temperature and humidity are more or less uniform horizontally for hundreds of kilometres i.e. a
body of air with horizontally uniform levels of temperature, humidity, and pressure is called as air
mass.
When the air remains over a homogenous area for a sufficiently long time, it acquires the
characteristics of the area. The homogenous regions can be the vast ocean surface or
vast plains. The homogenous surfaces, over which air masses form, are called the
source regions.
An ideal source region of air mass must possess the following essential conditions
• There must be extensive and homogeneous Earth’s surface so that it may possess
uniform temperature and moisture conditions. The source region should be either land
surface or ocean surface because irregular topography and surface comprised of both land
and water cannot have uniform temperature and moisture conditions. So, statement 1 is
correct.
• There should not be a convergence of air; instead, there should be a divergence of
airflow so that the air may stay over the region for a longer period and thus the air may
attain the physical properties of the region. It is, thus, apparent that anti-cyclonic areas
characterized by high barometric pressure and low-pressure gradients are most ideal
regions for the development of air masses. So, statement 2 is not correct.
• Atmospheric conditions should be stable for a considerably long period so that the air
may attain the characteristics of the surface. So, statement 3 is correct.
Therefore, the correct answer is (c).

Q77. Answer: (b)


Explanation:
Wyomesh Chandra Bonnerjee presided over the first session of the Indian National Congress
held at Bombay in 1885 from 28 December to 31 December. He was the president of the Indian
National Congress again in the 1892 session in Allahabad where he denounced the position
that India had to prove for the worthiness of political freedom. So, pair 1 is not correctly
matched.
George Yule (1829-1892) was a merchant in England and India who served as the fourth
President of the Indian National Congress in 1888 at Allahabad, the first non-Indian to hold
that office. So, pair 3 is not correctly matched.
Annie Besant was the first women president of INC session in1917. When World War I broke
out in 1914, she helped launch the Home Rule League to campaign for democracy in India, and
dominion status within the British Empire. This led to her election as president of the Indian
National Congress, in late 1917. So, pair 2 is correctly matched.
Therefore, the correct answer is (b).

Q78. Answer: (a)


Explanation: The duties, powers and functions of the Commissioner for Linguistic Minorities in
India have not been defined under any Statute except the Constitutional mandate under Article
350B.So, statement 1 is correct.
The Commissioner falls under the Ministry of Minority Affairs and on which he is dependent for
complete functioning. It shall be the duty of the Special Officer to investigate all matters relating
to the safeguards provided for linguistic minorities under this Constitution and report to the
President upon those matters at such intervals as the President may direct, and the President
shall cause all such reports to be laid before each House of Parliament, and sent to the
Governments of the States concerned. He submits those reports through the Union Minority
Affairs Minister. So, statement 2 is correct.
The Scheme of Safeguards for Linguistic Minorities adopted in the Chief Ministers Conference,
1961 have not yet been codified and cannot be enforced upon the States by the Commissioner.
So, statement 3 is not correct.

Prelim Test Series (2020) – GS Test 5 (12th April 2020)


Therefore, the correct answer is (a).

Q79. Answer: (b)


Explanation:
FDI is prohibited in India in the following sectors:
• Lottery Business, including Government/private lottery, online lotteries, etc.
• Gambling and Betting including casinos
• Chit Funds
• Nidhi Company
• Trading in Transferable Development Rights (TDR)
• Real Estate Business or Construction of farmhouses. Real estate business shall not
include the development of town shops, construction of residential/ commercial premises,
roads or bridges and Real Estate Investment Trusts (REITs) registered and regulated under
the SEBI (REITs) Regulations, 2014.
• Manufacturing of cigars, cheroots, cigarillos and cigarettes, of tobacco or tobacco
substitutes
• Sectors not open to private sector investment- atomic energy, railway operations (other than
permitted activities mentioned under the Consolidated FDI policy).
So, statements 3 and 4 are correct.
49% FDI is allowed in Private Security Agencies under the ‘automatic’ route. FDI above 49%
requires Governmental approval. So, statement 1 is not correct.
India permits foreign direct investment (FDI) in the multi-brand retail sector with a cap of 51 per
cent ownership by overseas players. However, the FDI policy also imposes several
conditions on foreign players related to mandatory sourcing of goods from MSMEs and
investment in back-end infrastructure. So, statement 2 is not correct.
Therefore, the correct answer is (b).

Q80. Answer: (c)


Explanation:
India has successfully achieved the complete phase-out of Hydrochlorofluorocarbon
(HCFC)-141 b. So, statement 1 is not correct.
It is a chemical used by foam manufacturing enterprises and is one of the most potent ozone-
depleting chemicals after Chlorofluorocarbons (CFCs). So, statement 2 is correct.
HCFC-141b is used mainly as a blowing agent in the production of rigid polyurethane (PU)
foams. So, statement 3 is correct.
Therefore, the correct answer is (c).

Q81. Answer: (b)


Explanation:
Soil Characteristics
1) Soil Texture: The texture of a soil refers to the sizes of the solid particles composing the
soil. Texture largely determines the water-retention properties of the soil. In sandy soil,
pore spaces are large, and water drains rapidly while in clay soil, the individual pore spaces
are too small for adequate drainage. Generally speaking, loam textures are best for plant
growth.
2) Soil Structure: Because of the cementing action of ions in the soil, individual particles in a
soil tend to aggregate together in lumps or peds. According to the shape of the peds, soils
can be described as having a blocky, platy, crumb, or prismatic structure. The soil structure
has an important bearing on its ease of cultivation. Soils with a crumb structure are best for
seed germination and are said to have good tilth.
3) Soil Colour: The colour of a soil depends upon its mineral and organic content. Dark brown
and reddish soils owe their colour to the presence of iron hydroxides. Dark soils,
approaching a black colour, are usually rich in carbon, generally in the form of decayed
organic matter.

Prelim Test Series (2020) – GS Test 5 (12th April 2020)


4) Soil Acidity and Alkalinity: Clay minerals have a complicated atomic structure and a vast
surface area in relation to their weight. Overall, they are negatively charged. This is
invariably neutralised by the attraction to their surfaces of positively-charged ions (cations)
of calcium, magnesium, potassium and sodium. These are known in soil science as bases.
They are only held loosely in an exchangeable position by the clay minerals and may be
given up in the process of base exchange to plants which require them for growth.
Under natural conditions, the bases are recycled to the soil by the decomposition of plants and
animals. Where the vegetation is removed by man by cutting or cropping, the bases can only be
fully replenished if supplied artificially in the form of fertiliser. Soft calcareous rocks are often
naturally fertile because the rate of weathering of the calcium in the parent material is sufficient
to replace the loss of leaching of exchangeable calcium. Lime helps to preserve structural
stability in soils. Thus, it determines the fertility of the soil.
Therefore, the correct answer is (b).

Q82. Answer: (a)


Explanation:
The Indian National Congress, which had been formed to create a mature political dialogue with
the British government, included both moderates and extremists.
When the Montagu report of 1918 was made public, there was a divide in the Congress over it.
The moderates welcomed it while the extremists opposed it. This led to a schism in the
Congress with moderate leaders forming the "Indian National Liberal Federation" in 1919. The
party (INLF) was founded by Surendra Nath Banerjee and some of its prominent leaders were
Tej Bahadur Sapru, V. S. Srinivasa Sastri and M. R. Jayakar. So, statement (a) is correct.
Therefore, the correct answer is (a).

Q83. Answer: (c)


Explanation: The original Constitution (Article 76) had provided for the office of the Attorney
General for India. He is the highest law officer in the country. The Attorney-General was
hence a part of the original constitution.
Originally, the Constitution of India did not make any provision with respect to the Special
Officer for Linguistic Minorities. Later, the States Reorganisation Commission (1953-55) made a
recommendation in this regard. Accordingly, the Seventh Constitutional Amendment Act of 1956
inserted a new Article 350-B in the constitution. Hence Special officer for linguistic
minorities was not a part of the original constitution
Originally, Article 338 of the Constitution provided for the appointment of a Special Officer for
Scheduled Castes (SCs) and Scheduled Tribes (STs) to investigate all matters relating to the
constitutional safeguards for the SCs and STs. The 65th Constitutional Amendment Act of 1990
provided for the establishment of a high-level multi-member National Commission for SCs and
STs in the place of a single Special Officer for SCs and STs. Later, the 89th Constitutional
Amendment Act of 2003 bifurcated the combined National Commission for SCs and STs into
two separate bodies the National Commission for Scheduled Castes and National Commission
for Scheduled Tribes. Hence both the National Commission for Scheduled Castes and the
National Commission for Scheduled Tribes were not a part of the original constitution.
Therefore, the answer is (c).

Q84. Answer: (d)


Explanation:
According to a study by the UN trade and investment body, India gained about $755 million in
additional exports, mainly of chemicals, metals and ore, to the US in the first half of 2019 due
to the trade diversion effects of Washington's tariff war with China. The study named 'Trade and
Trade Diversion Effects of United States Tariffs on China' shows that the ongoing US-China
trade war has resulted in a sharp decline in bilateral trade, higher prices for consumers and
trade diversion effects - increased imports from countries not directly involved in the trade war.
So, statement 1 is correct.

Prelim Test Series (2020) – GS Test 5 (12th April 2020)


Apart from trade, diversion in investment flows is an opportunity that India could benefit from, as
manufacturers seek alternative origination destinations. US FDI into India jumped in 2018,
accounting for 6 per cent of total investment flows. There has also been a notable pick-up in
flows from China. Larger gains are likely in the medium-term as India continues to work on
easing FDI regulations. So, statement 2 is correct.
According to a Bloomberg Economics report, uncertainty over trade could lower world gross
domestic product by 0.6% in 2021 compared to a no-trade-war scenario. So, statement 3 is
correct.
Recently, the United States designated China as a “currency manipulator”. The move came
immediately after the People’s Bank of China (PBoC), let the yuan weaken past the
psychologically significant CNY7 (CNY or the yuan is the basic unit of the renminbi, China’s
official currency) to the dollar mark. The yuan, was last at this level against the dollar more than
10 years ago in April 2008. The present devaluation of the currency has gained significance in
light of the ongoing trade war between the U.S. and China. If the U.S. weakens the dollar to
retaliate against China’s yuan devaluation, it will enter a currency war. So, statement 4 is
correct.
Therefore, the correct answer is (d).
Q85. Answer: (c)
Explanation: Recently, the Indian Space Research Organization has decided to set up five
Space Technology Cells at the premier institutes like IIT Bombay, Kanpur, Kharagpur and
Madras, IISc, Bengaluru and Joint Research Programme with Savitribai Phule, Pune University
(SPPU) to carry out research activities.
The Space Technology cells are being established under the RESPOND Programme. The
Technology cells will consist of professors and scholars associated with the institute. Under the
programme, the experts will conduct space technology research and applications. So,
statement 1 is correct.
The ISRO had started the RESPOND (Sponsored Research) programme in the 1970s. The
objective of the programme is to encourage academia to participate and contribute to various
Space-related research activities. Space Technology Cells. Under the programme, projects are
taken up by Universities/Academic Institutions in the areas of relevance to Space Programme.
ISRO helps these institutions to establish the necessary technical facilities and also provide
fellowships to researchers to work on cutting edge research topics. The programme is mutually
beneficial to both ISRO and Academia wherein the rich talent of Academia/ faculty is being
harnessed to support the Nation’s Space programme. So, statement 2 is correct.
Therefore, the correct answer is (c).

Q86. Answer: (c)


Explanation:
The Grand Banks of Newfoundland are a series of underwater plateaus south-east of the
island of Newfoundland on the North American continental shelf. The Grand Banks is one of the
world's richest fishing grounds, supporting Atlantic cod, swordfish, haddock and capelin, as well
as shellfish, seabirds and sea mammals. The cold Labrador Current mixes with the warm
waters of the Gulf Stream here, often causing extreme foggy conditions. So, pair 1 is correctly
matched.
Dogger Bank is a large sandbank in a shallow area of the North Sea about 100 kilometres off
the east coast of England. So, pair 2 is correctly matched.
Georges Bank is a large elevated area of the seafloor between Cape Cod, Massachusetts
(United States), and Cape Sable Island, Nova Scotia (Canada). It separates the Gulf of Maine
from the Atlantic Ocean. So, pair 3 is not matched correctly.
Therefore, the correct answer is (c).

Q87. Answer: (b)


Explanation:

Prelim Test Series (2020) – GS Test 5 (12th April 2020)


Bande Mataram was an English newspaper edited by Sri Aurobindo. The journal declared and
developed a new political programme for the country as the programme of the Nationalist Party,
non-cooperation, passive resistance, Swadeshi, Boycott, national education, settlement of
disputes in law by popular arbitration. Sri Aurobindo published in the paper a series of articles
on passive resistance, another developing a political philosophy of revolution and wrote many
leaders aimed at destroying the arguments of the Moderate Party. So, statement 1 is correct.
A number of revolutionaries started a plethora of newspapers to ignite the minds of the masses
and to enthuse them. Barindra Kumar Ghosh started a newspaper called Yugantar. The editor
of Yugantar, B.N Dutt had to face trial for allegedly anti-British activities. Shyamji Krishna Verma
published from London Indian Sociologist. It dispersed information of the Indian freedom
movement-related activities that occurred in England notably at the India House. Sandhya was
another publication which was fiercely critical of British policies. A number of editors had cases
filed against them and were tried for treason and sedition. So, statements 2 and 3 are correct.
In 1898 Shivram Mahadev Paranjape founded a weekly called Kal, which eventually would have
two meanings in Marathi; the first would be 'Times' and the other 'Terminator'. His fiery editorials
and humorous and sarcastic style of writing started making the younger generation furious with
anger against the British rule. Paranjape was arrested in 1908. British authorities convicted him
of “sedition” In 1910, when he was set free, British authorities banned the publication of Kal and
also confiscated collections of his past essays and editorials in the weekly. So, statement 4 is
correct.
Therefore, the correct answer is (b).

Q88. Answer: (b)


Explanation: There are certain lacunae with the Lokpal Act passed in 2013; these are :
• The complaint against corruption cannot be registered after a period of seven years from the
date on which the offence mentioned in such complaint is alleged to have been committed.
So, statement 3 is not correct.
• The judiciary has been excluded from the ambit of the Lokpal.
• Lokpal cannot suo-motu proceed against any public servant. So, statement 1 is correct.
• Heavy punishment for false and frivolous complaints against public servants
• Anonymous complaints not allowed – Hence one can’t just make a complaint on plain paper
and drop it in a box with supporting documents as can be done in the case of PIL’s. So,
statement 2 is correct.
• Very non-transparent procedure for dealing with complaints against the Prime Minister.
Therefore, the correct answer is (b).

Q89. Answer: (b)


Explanation:
World Economic Forum's Re-skilling Revolution is an initiative to provide one billion people with
better education, skills and jobs by 2030. The scheme aims to future-proof workers from
technological change and help economies by providing new skills for the Fourth
Industrial Revolution. Founding governments include Brazil, France, India, Pakistan, the
Russian Federation, UAE and the US. Business partners include PwC, Salesforce, Manpower
Group, Infosys, LinkedIn, Coursera Inc. and the Adecco Group. The Re-skilling Revolution
Platform aims to provide better jobs, education, and skills to 1 billion people in the next 10
years. This initiative will contribute to future-proofing countries, companies, and workforces in
order to build a fairer, more inclusive world that will deliver benefits to the economy and society
for generations to come. So, statement (b) is correct and statements (a), (c) and (d) are not
correct.
Therefore, the correct answer is (b).

Q90. Answer: (a)


Explanation:

Prelim Test Series (2020) – GS Test 5 (12th April 2020)


The World Economic Forum and its partners launched the 1t.org initiative. It is a multi-
stakeholder effort to grow, restore and conserve 1 trillion trees around the world. It is a mass
scale nature restoration initiative, as an essential part of the solution for tackling climate change
while also delivering important biodiversity, livelihood and broader sustainable development
benefits. So, statement 1 is correct.
The 1t.org project aims to unite governments, non-governmental organisations, businesses and
individuals for this "mass-scale nature restoration" initiative. So, statement 2 is correct.
1t.org is a World Economic Forum initiative, designed to support the UN Decade on Ecosystem
Restoration 2021-2030, led by UNEP and FAO. So, statement 3 is correct.
Therefore, the correct answer is (a).

Q91. Answer: (b)


Explanation:
The All India States Peoples' Conference (AISPC) was established by subjects of princely
states. It was a conglomeration of political movements in the princely states of the British Raj,
which were variously called Praja Mandals or Lok Parishads. The Conference brought together
representatives from hundreds of Indian princely states, including Baroda, Bhopal, Travancore
and Hyderabad. So, statement 1 is not correct.
The first session of the organisation was held in Bombay in December 1927. The Conference
looked to the Indian National Congress for support, but Congress was reluctant to provide it until
1939, when Jawaharlal Nehru became its president, serving in this position till 1946. So,
statement 2 is correct.
It was established to encourage political dialogue between the princely class of India, and the
British Raj, upon the issues of governance, political stability and future of India. For a long
period, the Conference was hostile to the Indian independence movement and often acted to
condemn and counter-act the work of the Indian National Congress when it was banned by
British authorities. So, statement 3 is correct.
Therefore, the correct answer is (b).

Q92. Answer: (c)


Explanation: The acts of Parliament or the state legislature do not apply to autonomous
districts and autonomous regions or apply with specified modifications and exceptions. The
power in this regard, lies either with the president or governor. In the case of Assam, it lies with
the Governor, both in respect of acts of Parliament or state legislature. In the case of
Meghalaya, Tripura and Mizoram, it lies with the president in respect of acts of Parliament and
governor in respect of acts of State legislature. So, statement 1 is correct.
The governor is empowered to organise and re-organise the autonomous districts. Thus, he can
increase or decrease their areas or change their names or define their boundaries, and so on.
Also, if there are different tribes in an autonomous district, the governor can divide the district
into several autonomous regions. So, statement 2 is correct.
Therefore, the correct answer is (c).

Q93. Answer: (d)


Explanation:
Autoclaving
The process of autoclaving involves steam sterilization. An autoclave is used to sterilize
surgical equipment, laboratory instruments, pharmaceutical items, and other materials.
Instead of incineration, which can be expensive, autoclaving simply introduces very hot steam
for a determined amount of time. At the end of the process, microorganisms will be completely
destroyed. This process is particularly effective because it costs much less than other methods,
and doesn’t present any personal health risks.

Prelim Test Series (2020) – GS Test 5 (12th April 2020)


Microwaving
During this process, waste is shredded, mixed with water, and then internally heated to kill
microorganisms and other harmful elements. One of the main benefits of this process is the
shredding aspect; it lowers the volume of biomedical waste, and it is reportedly more energy
efficient to use this method than to incinerate. While it can’t be used for all biomedical wastes, it
can be utilized for a good 90% of it, just like autoclaving.
Therefore, the correct answer is (d).

Q94. Answer: (b)


Explanation:
The Regional Comprehensive Economic Partnership (RCEP) is a proposed agreement between
the member states of the Association of Southeast Asian Nations (ASEAN) and its free trade
agreement (FTA) partners. The pact aims to cover trade in goods and services, intellectual
property, etc. The Regional Comprehensive Economic Partnership was introduced during the
19th ASEAN meeting held in November 2011. The RCEP negotiations were kick-started during
the 21st ASEAN Summit in Cambodia in November 2012. Member states of ASEAN and their
FTA partners are Brunei, Cambodia, Indonesia, Laos, Malaysia, Myanmar, the Philippines,
Singapore, Thailand, Vietnam, China, Japan, India, South Korea, Australia and New Zealand.
India did not join the Regional Comprehensive Economic Partnership (RCEP) because
concerns about getting swamped by imports under the agreement — putting its domestic
industry and agriculture at risk — were not assuaged.
India runs a large trade deficit with RCEP countries and was looking for specific protection
for its industry and farmers from a surge in imports, especially from China. India’s trade deficit
with the RCEP nations is $105 billion, of which China alone accounts for $54 billion. The main
worry is over Chinese manufactured goods and dairy products from New Zealand flooding
Indian markets, hurting domestic interests. The trade agreement was also seen as being
detrimental to the government’s Make in India initiative. So, statements (a) and (c) are
correct.
There was no concern pertaining to Indian investors investing in RCEP countries instead
of India. So, statement (b) is not correct.
There was also the issue of data localisation under RCEP deal. India wanted all countries to
have the rights to protect data. This would imply that countries can share data only where it is
“necessary to achieve a legitimate public policy objective” or "necessary in the country's opinion,
for the protection of its essential security interests or national interests". So, statement (d) is
correct.
Therefore, the answer is (b).

Q95. Answer: (a)


Explanation:
Muddiman Committee was appointed by the Government of India at the beginning of 1924.It
was established for making an investigation into the working of the constitution as set up in
1921 under the India Act of 1919. The committee was set up due to the growing political unrest
on the dyarchy issue of the Constitution. The committee was also known as the Reforms
Enquiry Committee. It had Sir Alexander Muddiman as the chairman, who served as the then
Home Member of the Government of India. So, statement (a) is correct.
Raleigh Commission (University Commission) was intended to make recommendations for
reforms of Universities of India. The recommendations enacted by Curzon as Indian University
Act, 1904. So, statement (b) is not correct.
Welby Commission was appointed to enquire income and expenditure of India. So, statement
(c) is not correct.
The Indian Currency Committee or Fowler Committee was a government committee appointed
by the British-run Government of India on 29 April 1898 to examine the currency situation in
India. So, statement (d) is not correct.
Therefore, the correct answer is (a).

Prelim Test Series (2020) – GS Test 5 (12th April 2020)


Q96. Answer: (a)
Explanation:
In a first, the Ministry of Environment, Forest and Climate Change, and the Centre for Media
Studies (CMS) organized VATAVARAN-2019, a Short Film Competition and Festival on
Environment. It was held in New Delhi. India was also the Chair of the 2019 Awards Jury of
CMS VATAVARAN. The festival is being organized, exclusively on environment and wildlife
along with the 10th competitive edition of CMS VATAVARAN.
Therefore, the correct answer is (a).

Q97. Answer: (b)


Explanation: There are certain restrictions on the Attorney General as given in the Law Officer
(Conditions of Service) Rules, 1987. The Attorney General shall not:
• hold briefs in any court for any party except the Government of India or the Government of a
State or any University, Government School or College, local authority, Public Service
Commission etc.
• advice any party against the Government of India or a Public Sector Undertaking
• defend an accused person in a criminal prosecution, without the permission of the
Government of India. So, option (a) is correct.
• accept appointment to any office in any company or corporation without the permission of
the Government of India. So, option(c) is correct.
• advice any Ministry or Department of Government of India or any statutory organization or
any Public Sector Undertaking unless the proposal or a reference in this regard is received
through the Ministry of Law and Justice, Department of Legal Affairs. So, option (d) is
correct.
Therefore, the answer is (b).

Q98. Answer: (a)


Explanation:
Biorock, also known as Seacrete/Seament, is a trademark name used by Biorock Inc. to refer to
the substance formed by electro-accumulation of minerals dissolved in seawater. Wolf Hilbertz
developed the process and patented it in 1979. The building process, popularly called accretion,
is not to be confused with Biorock sewage treatment. The biorock building process grows
cement-like engineering structures and marine ecosystems, often for mariculture of corals,
oysters, clams, lobsters and fish in saltwater. It works by passing a small electric current
through electrodes in the water. The structure grows more or less without limit as long as
current flows.
BENEFITS OF BIOROCK:
Biorock accelerates growth on coral reefs by as much as fivefold and restoration of physical
damage by as much as 20 times. When mixed with construction aggregates, it can build
components on the seafloor or land. Biorock represents the only known method that can sustain
and grow natural coral species using only basic conducting elements, typically of a common
metal such as steel.
Biorock reefs grow rapidly and strengthen as they age. They thus have great potential for many
applications, such as making breakwaters.
Biorock is cost-effective, requiring only metal bars or equivalent and electricity. While electricity
provided from fossil fuels generates CO2, bio rock projects often use renewable solar power,
wind power, tidal power, or wave power. The resulting material is cheaper than concrete blocks
in many places, depending on electricity and cement transport costs.
Therefore, the correct answer is (a).

Q99. Answer: (b)


Explanation: The ASEAN Regional Forum (ARF) was set up in 1993 as regional security
cooperation and dialogue platform based on deliberations in the ASEAN Post Ministerial

Prelim Test Series (2020) – GS Test 5 (12th April 2020)


Conference between the Foreign Ministers of ASEAN and its full dialogue partners. In 1995, the
ARF agreed to an evolutionary approach to effectively tackle security issues and adopted
a 3 stage process viz, 1st stage Confidence Building Measures (CBMs); 2nd stage-
development of preventive diplomacy; and 3rd stage- elaboration of approaches to
conflicts. ASEAN values of consensus, confidence-building and progress at a pace
comfortable to all have guided the ARF process since inception. At present, ARF has 27
members.
Therefore, the correct answer is (b).

Q100. Answer: (a)


Explanation:
In 1924, Bahishkrit Hitakarini Sabha was formed by Dr. Ambedkar for removing difficulties of the
untouchables and placing their grievances before government. This was his first organization to
achieve his political and social ideals. To promote the spread of education among the
Depressed Class and to advance and improve the economic condition of the Depressed
Classes by starting Industrial and Agricultural schools were some of its objectives. So,
statement 1 is correct.
Gandhi founded All India Anti Untouchability League, to remove untouchability in the society,
which later renamed as Harijan Sevak Sangh ("Servants of Untouchables Society"). At the time
industrialist Ghanshyam Das Birla was its founding president with Amritlal Takkar as its
Secretary. So, statement 2 is not correct.
B. R. Ambedkar led a protest outside Kalaram temple (in Nashik) in 1930, in order to allow
Dalits into the temple. It was one of the non-violent movement where around 15000 people had
participated in getting entry into the temple. This Satyagraha continued for 5 years, and finally,
the temple gates were opened for the untouchables. Ambedkar, in his speech on inauguration
of Satyagraha said “We don’t want to go to temples though, but we should have rights.” So,
statement 3 is correct.
Therefore, the correct answer is (a).

Prelim Test Series (2020) – GS Test 5 (12th April 2020)

You might also like